Let $h_a, h_b, h_c$ be the lengths of the altitudes of a triangle $ABC$ from $A, B, C$ respectively. Let $P$ be any point inside the triangle. Show that \[\frac{PA}{h_b+h_c} + \frac{PB}{h_a+h_c} + \frac{PC}{h_a+h_b} \ge 1.\]
Problem
Source:
Tags: inequalities, geometry, circumcircle, geometric transformation, reflection, trigonometry, triangle inequality
28.07.2010 20:07
Note: This is not a complete solution, but hopefully it's on the right track... For the hard case (subcase 2.2) I have only been able to find the optimal $P$. (For more on the "weighted/generalized Fermat point," see here and here.)
01.08.2010 14:46
A well-known lemma used in proving the Erdos-Mordell inequality kills this problem. The lemma statement is this: [Lemma] Let $P$ be a point inside triangle $ABC$, and let the projections of $P$ onto $BC,CA,AB$ be $D,E,F$. Denote $BC = a$, $CA=b$, $AB=c$ be the lengths of the three sides. Then we have the following two inequalities: Inequality 1: $a \cdot PA \geq PE \cdot b + PF \cdot c$ Inequality 2: $a \cdot PA \geq PE \cdot c + PF \cdot b$ The first can be proved in the following way: denote the projection of $A$ onto $BC$ by $H$. Letting the area of $ABC$ be $S$, we have $a \cdot PD + b \cdot PE + c \cdot PF = 2S = a \cdot AH \leq a \cdot (AP + PD)$ which yields Inequality 1. The second is a consequence of the first, this is obvious once you reflect $P$ wrt the internal bisector of angle $A$. Once you know these two inequalities, adding them and applying the (added) inequality to the original inequality reduces the TST problem to an AM-GM exercise. In fact, the same lemma kills a former USA TST problem: http://www.artofproblemsolving.com/Forum/viewtopic.php?p=20670&sid=a8f03b9f6416bf2325927e84a7a3493e#p20670
27.10.2010 07:15
25.02.2015 18:08
I have found a false solution that I would just like to show quickly to highlight how strong the inequality is. We can reduce stuff using $h_a=b\sin C$ and $\frac{a}{\sin A}=2R$ and it becomes $\sum \frac{PA}{ab+ac}\ge \frac{1}{2R}$. With Cauchy it suffices to show that $\left( \sum \sqrt{PA}\right) ^2\ge \frac{ab+bc+ca}{R}.$ Now the estimate $9R\ge ab+bc+ca$ is true, but is very tight. It falls to SOS as $\sum (b-c)^2bc(2b^2+4bc+2c^2+ab+ac-3a^2)$, which is still very tight when $c$ is small, but whatever it dies. So it reduces to $\sum \sqrt{PA} \ge 3\sqrt{R}$ which is very wrong when $a\approx b+c$. If I find a solution along these lines I'll post it.
23.07.2021 04:46
Similar to the above solutions
30.05.2023 21:25